Difference between revisions of "2007 AMC 8 Problems/Problem 15"

(Solution)
m (Solution 2)
 
(13 intermediate revisions by 9 users not shown)
Line 4: Line 4:
 
impossible?
 
impossible?
  
<math>\mathrm{(A)} \ a + c < b  \qquad \mathrm{(B)} \ a * b < c \qquad \mathrm{(C)} \ a + b < c \qquad \mathrm{(D)} \ a * c < b \qquad \mathrm{(E)}\frac{b}{c} = a</math>
+
<math>\mathrm{(A)} \ a + c < b  \qquad \mathrm{(B)} \ a \cdot b < c \qquad \mathrm{(C)} \ a + b < c \qquad \mathrm{(D)} \ a \cdot c < b \qquad \mathrm{(E)}\frac{b}{c} = a</math>
  
 
== Solution ==
 
== Solution ==
  
According to the given rules,  
+
According to the given rules, every number needs to be positive. Since <math>c</math> is always greater than <math>b</math>, adding a positive number (<math>a</math>) to <math>c</math> will always make it greater than <math>b</math>.
  
Every number needs to be positive.
+
Therefore, the answer is <math>\boxed{\textbf{(A)}\ a+c<b}</math>
  
Since <math>c</math> is always greater than <math>b</math>,
 
  
adding a positive number (<math>a</math>) to <math>c</math> will always make it greater than <math>b</math>.
+
==Solution 2==
 +
 
 +
We can test numbers into the inequality we’re given. The simplest is <math>0<1<2<3</math>. We can see that <math>3+1>2</math>, so <math>\boxed{\textbf{(A) }a+c<b}</math> is correct.
 +
 
 +
—jason.ca
 +
 
 +
==Video Solution by WhyMath==
 +
https://youtu.be/UdzJetT-XOY
 +
 
 +
~savannahsolver
 +
 
 +
==Video Solution==
 +
 
 +
https://www.youtube.com/watch?v=_ZHS4M7kpnE
 +
 
 +
==Video Solution 2==
 +
https://youtu.be/GxR1giTQeD0  Soo, DRMS, NM
  
Therefore, the answer is <math>\boxed{A}</math>
 
  
 
==See Also==
 
==See Also==
 
{{AMC8 box|year=2007|num-b=14|num-a=16}}
 
{{AMC8 box|year=2007|num-b=14|num-a=16}}
 +
{{MAA Notice}}

Latest revision as of 17:18, 3 November 2023

Problem

Let $a, b$ and $c$ be numbers with $0 < a < b < c$. Which of the following is impossible?

$\mathrm{(A)} \ a + c < b  \qquad \mathrm{(B)} \ a \cdot b < c \qquad \mathrm{(C)} \ a + b < c \qquad \mathrm{(D)} \ a \cdot c < b \qquad \mathrm{(E)}\frac{b}{c} = a$

Solution

According to the given rules, every number needs to be positive. Since $c$ is always greater than $b$, adding a positive number ($a$) to $c$ will always make it greater than $b$.

Therefore, the answer is $\boxed{\textbf{(A)}\ a+c<b}$


Solution 2

We can test numbers into the inequality we’re given. The simplest is $0<1<2<3$. We can see that $3+1>2$, so $\boxed{\textbf{(A) }a+c<b}$ is correct.

—jason.ca

Video Solution by WhyMath

https://youtu.be/UdzJetT-XOY

~savannahsolver

Video Solution

https://www.youtube.com/watch?v=_ZHS4M7kpnE

Video Solution 2

https://youtu.be/GxR1giTQeD0 Soo, DRMS, NM


See Also

2007 AMC 8 (ProblemsAnswer KeyResources)
Preceded by
Problem 14
Followed by
Problem 16
1 2 3 4 5 6 7 8 9 10 11 12 13 14 15 16 17 18 19 20 21 22 23 24 25
All AJHSME/AMC 8 Problems and Solutions

The problems on this page are copyrighted by the Mathematical Association of America's American Mathematics Competitions. AMC logo.png